Difference between revisions of "2002 AMC 12A Problems/Problem 2"

(New page: ==Problem== {{empty}} ==Solution== {{solution}} ==See Also== {{AMC12 box|year=2002|ab=A|num-b=1|num-a=3}})
 
(3 intermediate revisions by 2 users not shown)
Line 1: Line 1:
 +
{{duplicate|[[2002 AMC 12A Problems|2002 AMC 12A #2]] and [[2002 AMC 10A Problems|2002 AMC 10A #6]]}}
 +
 
==Problem==
 
==Problem==
  
{{empty}}
+
Cindy was asked by her teacher to subtract 3 from a certain number and then divide the result by 9. Instead, she subtracted 9 and then divided the result by 3, giving an answer of 43. What would her answer have been had she worked the problem correctly?
 +
 
 +
<math> \mathrm{(A) \ } 15\qquad \mathrm{(B) \ } 34\qquad \mathrm{(C) \ } 43\qquad \mathrm{(D) \ } 51\qquad \mathrm{(E) \ } 138 </math>
 +
 
  
 
==Solution==
 
==Solution==
  
{{solution}}
+
We work backwards; the number that Cindy started with is <math>3(43)+9=138</math>. Now, the correct result is <math>\frac{138-3}{9}=\frac{135}{9}=15</math>. Our answer is <math>\boxed{\text{(A)}\ 15}</math>.
  
 
==See Also==
 
==See Also==
  
 
{{AMC12 box|year=2002|ab=A|num-b=1|num-a=3}}
 
{{AMC12 box|year=2002|ab=A|num-b=1|num-a=3}}
 +
{{AMC10 box|year=2002|ab=A|num-b=5|num-a=7}}
 +
{{MAA Notice}}

Revision as of 10:06, 4 July 2013

The following problem is from both the 2002 AMC 12A #2 and 2002 AMC 10A #6, so both problems redirect to this page.

Problem

Cindy was asked by her teacher to subtract 3 from a certain number and then divide the result by 9. Instead, she subtracted 9 and then divided the result by 3, giving an answer of 43. What would her answer have been had she worked the problem correctly?

$\mathrm{(A) \ } 15\qquad \mathrm{(B) \ } 34\qquad \mathrm{(C) \ } 43\qquad \mathrm{(D) \ } 51\qquad \mathrm{(E) \ } 138$


Solution

We work backwards; the number that Cindy started with is $3(43)+9=138$. Now, the correct result is $\frac{138-3}{9}=\frac{135}{9}=15$. Our answer is $\boxed{\text{(A)}\ 15}$.

See Also

2002 AMC 12A (ProblemsAnswer KeyResources)
Preceded by
Problem 1
Followed by
Problem 3
1 2 3 4 5 6 7 8 9 10 11 12 13 14 15 16 17 18 19 20 21 22 23 24 25
All AMC 12 Problems and Solutions
2002 AMC 10A (ProblemsAnswer KeyResources)
Preceded by
Problem 5
Followed by
Problem 7
1 2 3 4 5 6 7 8 9 10 11 12 13 14 15 16 17 18 19 20 21 22 23 24 25
All AMC 10 Problems and Solutions

The problems on this page are copyrighted by the Mathematical Association of America's American Mathematics Competitions. AMC logo.png